PT75.S3.Q22 - From 1996 to 2004, the average...

Rev_LefeRev_Lefe Member
edited May 2021 in Logical Reasoning 385 karma

Could someone please explain further about E?

In terms of (E), how do we know that "the biggest decreases in family income" were occurred within the period from 1996 to 2004? Because of this trouble, I picked E under the concept that I can't make an unwarranted assumption about the time the biggest decreases occurred. I can't really figure out what did I miss.

Thank you for your help.

Admin Note: https://7sage.com/lsat_explanations/lsat-75-section-3-question-22/

Comments

  • Jonathan WangJonathan Wang Yearly Sage
    edited May 2021 6867 karma

    It's a weaken (except) question. When you manipulate arguments, you're not supposed to question the answer choice's truth, you're supposed to plug it into the argument to ascertain its impact ("which of the following, if true...").

    Plugged in, the answer choice takes the blame off of the ruling party, attacking the idea that the decreases are the ruling party's fault.

Sign In or Register to comment.